Difference between revisions of "2022 AMC 12B Problems/Problem 16"
(→Solution 2) |
m (→Solution 3) |
||
Line 37: | Line 37: | ||
Let <math>x = 2^a, y = 2^b.</math> We have <math>(2^a)^{2^b} = 2^{64} \Rightarrow 2^{a\cdot 2^b} = 2^{64} \Rightarrow a\cdot 2^b = 64,</math> and <math>a^b = 128</math>. | Let <math>x = 2^a, y = 2^b.</math> We have <math>(2^a)^{2^b} = 2^{64} \Rightarrow 2^{a\cdot 2^b} = 2^{64} \Rightarrow a\cdot 2^b = 64,</math> and <math>a^b = 128</math>. | ||
− | Then, from eq 1, <math>a = 64\cdot 2^{-b},</math> and substituting in to eq 2, <math>(64\cdot 2^{-b})^b = 64^b\cdot 2^{-b^2} = 2^6b\cdot 2^{-b^2} = 2^{6b-b^2} = 2^{7}.</math> Thus, <math>6b-b^2 = 7.</math> | + | Then, from eq 1, <math>a = 64\cdot 2^{-b},</math> and substituting in to eq 2, <math>(64\cdot 2^{-b})^b = 64^b\cdot 2^{-b^2} = 2^{6b}\cdot 2^{-b^2} = 2^{6b-b^2} = 2^{7}.</math> Thus, <math>6b-b^2 = 7.</math> |
Solving for <math>b</math> using the quadratic formula gets <math>b = 3 \pm \sqrt{2}.</math> Since we are looking for <math>\log_2{y}</math> which equals <math>b,</math> we put <math>\boxed{\textbf{(C)} \ 3+\sqrt{2}}</math> as our answer. | Solving for <math>b</math> using the quadratic formula gets <math>b = 3 \pm \sqrt{2}.</math> Since we are looking for <math>\log_2{y}</math> which equals <math>b,</math> we put <math>\boxed{\textbf{(C)} \ 3+\sqrt{2}}</math> as our answer. |
Revision as of 21:29, 20 November 2022
Problem
Suppose and are positive real numbers such that
and .
What is the greatest possible value of ?
Solution
Take the base-two logarithm of both equations to get Now taking the base-two logarithm of the first equation again yields It follows that the real numbers and satisfy and . Solving this system yields Thus the largest possible value of is .
cr. djmathman
Solution 2
.
Substitution into yields
.
Solving for yields or , and we take the greater value .
~4SunnyH
Solution 3
Let We have and .
Then, from eq 1, and substituting in to eq 2, Thus,
Solving for using the quadratic formula gets Since we are looking for which equals we put as our answer.
~sirswagger21
See Also
2022 AMC 12B (Problems • Answer Key • Resources) | |
Preceded by Problem 15 |
Followed by Problem 17 |
1 • 2 • 3 • 4 • 5 • 6 • 7 • 8 • 9 • 10 • 11 • 12 • 13 • 14 • 15 • 16 • 17 • 18 • 19 • 20 • 21 • 22 • 23 • 24 • 25 | |
All AMC 12 Problems and Solutions |
The problems on this page are copyrighted by the Mathematical Association of America's American Mathematics Competitions.